what is the value of the current (i )and what is the right way to do it?












3














can someone please point out what exactly went wrong with the below method.



here,in a same circuit I am getting to different value for the current in the mid wire only by rearranging the circuit.



image 1



I was trying to solve this unbalanced Wheatstone bridge and found that the current (i) in the mid wire (in image 1) is 3 amp .This result was achieved by using node voltage method.



Then I rearranged the same circuit to the form shown in the second image.



image 2



and at this point I flipped the right half of the circuit so as to obtain the equal resistors on the same side.



iamge 3



now I again rearranged the circuit as shown.



image 4



Here, the final circuit is a balanced Wheatstone bridge and thus the current (i) must be 0 amp



So clearly something went wrong in there.



I posted a question before this one just to make sure that there is no mistake in rearranging the circuit in the way I have done.



THE MID WIRE IS A COMPLETE CONNECTED WIRE . THE GAP IN THE MID WIRE NEAR THE ARROW WAS INEVITABLE










share|improve this question







New contributor




Feels awesome is a new contributor to this site. Take care in asking for clarification, commenting, and answering.
Check out our Code of Conduct.
















  • 1




    this question was at first asked in physics stack exchange by me but it was unfortunately put on hold. link of first question -- physics.stackexchange.com/questions/450822/…
    – Feels awesome
    3 hours ago












  • Didn't we just go through all this, like a day ago?
    – jonk
    1 hour ago










  • that was a kind guy who posted my question again as the one that i posted was put on hold of some unknown reason . as the qs he posted has some errors I posted it again .
    – Feels awesome
    55 mins ago










  • Possible duplicate of What's wrong with this Wheatstone bridge?
    – fhlb
    51 mins ago










  • ya I have posted it again to get better ans
    – Feels awesome
    48 mins ago
















3














can someone please point out what exactly went wrong with the below method.



here,in a same circuit I am getting to different value for the current in the mid wire only by rearranging the circuit.



image 1



I was trying to solve this unbalanced Wheatstone bridge and found that the current (i) in the mid wire (in image 1) is 3 amp .This result was achieved by using node voltage method.



Then I rearranged the same circuit to the form shown in the second image.



image 2



and at this point I flipped the right half of the circuit so as to obtain the equal resistors on the same side.



iamge 3



now I again rearranged the circuit as shown.



image 4



Here, the final circuit is a balanced Wheatstone bridge and thus the current (i) must be 0 amp



So clearly something went wrong in there.



I posted a question before this one just to make sure that there is no mistake in rearranging the circuit in the way I have done.



THE MID WIRE IS A COMPLETE CONNECTED WIRE . THE GAP IN THE MID WIRE NEAR THE ARROW WAS INEVITABLE










share|improve this question







New contributor




Feels awesome is a new contributor to this site. Take care in asking for clarification, commenting, and answering.
Check out our Code of Conduct.
















  • 1




    this question was at first asked in physics stack exchange by me but it was unfortunately put on hold. link of first question -- physics.stackexchange.com/questions/450822/…
    – Feels awesome
    3 hours ago












  • Didn't we just go through all this, like a day ago?
    – jonk
    1 hour ago










  • that was a kind guy who posted my question again as the one that i posted was put on hold of some unknown reason . as the qs he posted has some errors I posted it again .
    – Feels awesome
    55 mins ago










  • Possible duplicate of What's wrong with this Wheatstone bridge?
    – fhlb
    51 mins ago










  • ya I have posted it again to get better ans
    – Feels awesome
    48 mins ago














3












3








3







can someone please point out what exactly went wrong with the below method.



here,in a same circuit I am getting to different value for the current in the mid wire only by rearranging the circuit.



image 1



I was trying to solve this unbalanced Wheatstone bridge and found that the current (i) in the mid wire (in image 1) is 3 amp .This result was achieved by using node voltage method.



Then I rearranged the same circuit to the form shown in the second image.



image 2



and at this point I flipped the right half of the circuit so as to obtain the equal resistors on the same side.



iamge 3



now I again rearranged the circuit as shown.



image 4



Here, the final circuit is a balanced Wheatstone bridge and thus the current (i) must be 0 amp



So clearly something went wrong in there.



I posted a question before this one just to make sure that there is no mistake in rearranging the circuit in the way I have done.



THE MID WIRE IS A COMPLETE CONNECTED WIRE . THE GAP IN THE MID WIRE NEAR THE ARROW WAS INEVITABLE










share|improve this question







New contributor




Feels awesome is a new contributor to this site. Take care in asking for clarification, commenting, and answering.
Check out our Code of Conduct.











can someone please point out what exactly went wrong with the below method.



here,in a same circuit I am getting to different value for the current in the mid wire only by rearranging the circuit.



image 1



I was trying to solve this unbalanced Wheatstone bridge and found that the current (i) in the mid wire (in image 1) is 3 amp .This result was achieved by using node voltage method.



Then I rearranged the same circuit to the form shown in the second image.



image 2



and at this point I flipped the right half of the circuit so as to obtain the equal resistors on the same side.



iamge 3



now I again rearranged the circuit as shown.



image 4



Here, the final circuit is a balanced Wheatstone bridge and thus the current (i) must be 0 amp



So clearly something went wrong in there.



I posted a question before this one just to make sure that there is no mistake in rearranging the circuit in the way I have done.



THE MID WIRE IS A COMPLETE CONNECTED WIRE . THE GAP IN THE MID WIRE NEAR THE ARROW WAS INEVITABLE







current circuit-analysis wheatstone-bridge nodal-analysis






share|improve this question







New contributor




Feels awesome is a new contributor to this site. Take care in asking for clarification, commenting, and answering.
Check out our Code of Conduct.











share|improve this question







New contributor




Feels awesome is a new contributor to this site. Take care in asking for clarification, commenting, and answering.
Check out our Code of Conduct.









share|improve this question




share|improve this question






New contributor




Feels awesome is a new contributor to this site. Take care in asking for clarification, commenting, and answering.
Check out our Code of Conduct.









asked 3 hours ago









Feels awesome

183




183




New contributor




Feels awesome is a new contributor to this site. Take care in asking for clarification, commenting, and answering.
Check out our Code of Conduct.





New contributor





Feels awesome is a new contributor to this site. Take care in asking for clarification, commenting, and answering.
Check out our Code of Conduct.






Feels awesome is a new contributor to this site. Take care in asking for clarification, commenting, and answering.
Check out our Code of Conduct.








  • 1




    this question was at first asked in physics stack exchange by me but it was unfortunately put on hold. link of first question -- physics.stackexchange.com/questions/450822/…
    – Feels awesome
    3 hours ago












  • Didn't we just go through all this, like a day ago?
    – jonk
    1 hour ago










  • that was a kind guy who posted my question again as the one that i posted was put on hold of some unknown reason . as the qs he posted has some errors I posted it again .
    – Feels awesome
    55 mins ago










  • Possible duplicate of What's wrong with this Wheatstone bridge?
    – fhlb
    51 mins ago










  • ya I have posted it again to get better ans
    – Feels awesome
    48 mins ago














  • 1




    this question was at first asked in physics stack exchange by me but it was unfortunately put on hold. link of first question -- physics.stackexchange.com/questions/450822/…
    – Feels awesome
    3 hours ago












  • Didn't we just go through all this, like a day ago?
    – jonk
    1 hour ago










  • that was a kind guy who posted my question again as the one that i posted was put on hold of some unknown reason . as the qs he posted has some errors I posted it again .
    – Feels awesome
    55 mins ago










  • Possible duplicate of What's wrong with this Wheatstone bridge?
    – fhlb
    51 mins ago










  • ya I have posted it again to get better ans
    – Feels awesome
    48 mins ago








1




1




this question was at first asked in physics stack exchange by me but it was unfortunately put on hold. link of first question -- physics.stackexchange.com/questions/450822/…
– Feels awesome
3 hours ago






this question was at first asked in physics stack exchange by me but it was unfortunately put on hold. link of first question -- physics.stackexchange.com/questions/450822/…
– Feels awesome
3 hours ago














Didn't we just go through all this, like a day ago?
– jonk
1 hour ago




Didn't we just go through all this, like a day ago?
– jonk
1 hour ago












that was a kind guy who posted my question again as the one that i posted was put on hold of some unknown reason . as the qs he posted has some errors I posted it again .
– Feels awesome
55 mins ago




that was a kind guy who posted my question again as the one that i posted was put on hold of some unknown reason . as the qs he posted has some errors I posted it again .
– Feels awesome
55 mins ago












Possible duplicate of What's wrong with this Wheatstone bridge?
– fhlb
51 mins ago




Possible duplicate of What's wrong with this Wheatstone bridge?
– fhlb
51 mins ago












ya I have posted it again to get better ans
– Feels awesome
48 mins ago




ya I have posted it again to get better ans
– Feels awesome
48 mins ago










3 Answers
3






active

oldest

votes


















1














You start with a primitive circuit, and then apply a series of re-configurations, becoming progressively more sophisticated, until you arrive at the final level of sophistication - a single 4 ohm resistor and a source.



Along the way, the currents and voltages in/at the various conductors and nodes that you introduce, change. It's not surprising. There are an infinite number of ways of transforming a single resistor into a bridge.



In 'simplifying' the original primitive bridge circuit you lose information that you cannot recover. You cannot rediscover the bridge if all you have to work on is a 36 volt source and a 4 ohm resistor.






share|improve this answer























  • so u that mean re-configuring the circuit is the mistake.
    – Feels awesome
    41 mins ago










  • Not at all. It depends what you're doing it for. You lose the detail of the circuit when you simplify, and you can't go on a random path and expect to recreate it.
    – Chu
    39 mins ago












  • now it makes sense . 👍
    – Feels awesome
    36 mins ago



















0














THe upper cct has a bridge current of 12V/4Ω=3A but a total current of 36V /4.5Ω = 8A from (9//9=4.5Ω)



The lower circuit has a bridge current of 0A yet still a total current of 8A.





schematic





simulate this circuit – Schematic created using CircuitLab



These are not the same circuits.






share|improve this answer























  • but hey both are same circuits ,right ?.and we still get two different values of current hiw is it possible?
    – Feels awesome
    2 hours ago










  • THey total current is the same but not the same circuits with the R's swapped and bridged. Once side is 12V the other 24V
    – Tony EE rocketscientist
    2 hours ago










  • and the upper circuit has the current of 3 amp in the bridge .
    – Feels awesome
    2 hours ago










  • typo...........
    – Tony EE rocketscientist
    2 hours ago










  • why arent they equivalent circuits ? I think , I have only rearranged them without any mistakes
    – Feels awesome
    2 hours ago



















0














Analyzing the circuit by using only Ohm's law and equivalent resistance we can calculate the current passing through each wire in the circuit:



circuit



It is clear that there's a 3A current going from node C to D.



Now if you switched R3 and R4 positions. then indeed no current will flow through the mid wire but the thing is, the current passing through the wire is not i. call it i2 as the circuit has indeed changed.






share|improve this answer





















  • i agree with ur point . but we know that the first circuit(in my qs) is equivalent to the second circuit(in my qs) . so there is most probably no mistake the rearrangement .so where is the mistake ?
    – Feels awesome
    1 hour ago












  • node C and node D dissappeared. so it's not the same circuit.
    – fhlb
    1 hour ago










  • no the node c and D has not disappeared it is present in the last circuit
    – Feels awesome
    59 mins ago










  • 2 new nodes appeared but C and D are clearly note there. C by definition is the node joining R2,R4 and node D
    – fhlb
    53 mins ago










  • thats fine but what is the error in my rearrangement . ?
    – Feels awesome
    51 mins ago











Your Answer





StackExchange.ifUsing("editor", function () {
return StackExchange.using("mathjaxEditing", function () {
StackExchange.MarkdownEditor.creationCallbacks.add(function (editor, postfix) {
StackExchange.mathjaxEditing.prepareWmdForMathJax(editor, postfix, [["\$", "\$"]]);
});
});
}, "mathjax-editing");

StackExchange.ifUsing("editor", function () {
return StackExchange.using("schematics", function () {
StackExchange.schematics.init();
});
}, "cicuitlab");

StackExchange.ready(function() {
var channelOptions = {
tags: "".split(" "),
id: "135"
};
initTagRenderer("".split(" "), "".split(" "), channelOptions);

StackExchange.using("externalEditor", function() {
// Have to fire editor after snippets, if snippets enabled
if (StackExchange.settings.snippets.snippetsEnabled) {
StackExchange.using("snippets", function() {
createEditor();
});
}
else {
createEditor();
}
});

function createEditor() {
StackExchange.prepareEditor({
heartbeatType: 'answer',
autoActivateHeartbeat: false,
convertImagesToLinks: false,
noModals: true,
showLowRepImageUploadWarning: true,
reputationToPostImages: null,
bindNavPrevention: true,
postfix: "",
imageUploader: {
brandingHtml: "Powered by u003ca class="icon-imgur-white" href="https://imgur.com/"u003eu003c/au003e",
contentPolicyHtml: "User contributions licensed under u003ca href="https://creativecommons.org/licenses/by-sa/3.0/"u003ecc by-sa 3.0 with attribution requiredu003c/au003e u003ca href="https://stackoverflow.com/legal/content-policy"u003e(content policy)u003c/au003e",
allowUrls: true
},
onDemand: true,
discardSelector: ".discard-answer"
,immediatelyShowMarkdownHelp:true
});


}
});






Feels awesome is a new contributor. Be nice, and check out our Code of Conduct.










draft saved

draft discarded


















StackExchange.ready(
function () {
StackExchange.openid.initPostLogin('.new-post-login', 'https%3a%2f%2felectronics.stackexchange.com%2fquestions%2f414259%2fwhat-is-the-value-of-the-current-i-and-what-is-the-right-way-to-do-it%23new-answer', 'question_page');
}
);

Post as a guest















Required, but never shown

























3 Answers
3






active

oldest

votes








3 Answers
3






active

oldest

votes









active

oldest

votes






active

oldest

votes









1














You start with a primitive circuit, and then apply a series of re-configurations, becoming progressively more sophisticated, until you arrive at the final level of sophistication - a single 4 ohm resistor and a source.



Along the way, the currents and voltages in/at the various conductors and nodes that you introduce, change. It's not surprising. There are an infinite number of ways of transforming a single resistor into a bridge.



In 'simplifying' the original primitive bridge circuit you lose information that you cannot recover. You cannot rediscover the bridge if all you have to work on is a 36 volt source and a 4 ohm resistor.






share|improve this answer























  • so u that mean re-configuring the circuit is the mistake.
    – Feels awesome
    41 mins ago










  • Not at all. It depends what you're doing it for. You lose the detail of the circuit when you simplify, and you can't go on a random path and expect to recreate it.
    – Chu
    39 mins ago












  • now it makes sense . 👍
    – Feels awesome
    36 mins ago
















1














You start with a primitive circuit, and then apply a series of re-configurations, becoming progressively more sophisticated, until you arrive at the final level of sophistication - a single 4 ohm resistor and a source.



Along the way, the currents and voltages in/at the various conductors and nodes that you introduce, change. It's not surprising. There are an infinite number of ways of transforming a single resistor into a bridge.



In 'simplifying' the original primitive bridge circuit you lose information that you cannot recover. You cannot rediscover the bridge if all you have to work on is a 36 volt source and a 4 ohm resistor.






share|improve this answer























  • so u that mean re-configuring the circuit is the mistake.
    – Feels awesome
    41 mins ago










  • Not at all. It depends what you're doing it for. You lose the detail of the circuit when you simplify, and you can't go on a random path and expect to recreate it.
    – Chu
    39 mins ago












  • now it makes sense . 👍
    – Feels awesome
    36 mins ago














1












1








1






You start with a primitive circuit, and then apply a series of re-configurations, becoming progressively more sophisticated, until you arrive at the final level of sophistication - a single 4 ohm resistor and a source.



Along the way, the currents and voltages in/at the various conductors and nodes that you introduce, change. It's not surprising. There are an infinite number of ways of transforming a single resistor into a bridge.



In 'simplifying' the original primitive bridge circuit you lose information that you cannot recover. You cannot rediscover the bridge if all you have to work on is a 36 volt source and a 4 ohm resistor.






share|improve this answer














You start with a primitive circuit, and then apply a series of re-configurations, becoming progressively more sophisticated, until you arrive at the final level of sophistication - a single 4 ohm resistor and a source.



Along the way, the currents and voltages in/at the various conductors and nodes that you introduce, change. It's not surprising. There are an infinite number of ways of transforming a single resistor into a bridge.



In 'simplifying' the original primitive bridge circuit you lose information that you cannot recover. You cannot rediscover the bridge if all you have to work on is a 36 volt source and a 4 ohm resistor.







share|improve this answer














share|improve this answer



share|improve this answer








edited 41 mins ago

























answered 44 mins ago









Chu

5,1052611




5,1052611












  • so u that mean re-configuring the circuit is the mistake.
    – Feels awesome
    41 mins ago










  • Not at all. It depends what you're doing it for. You lose the detail of the circuit when you simplify, and you can't go on a random path and expect to recreate it.
    – Chu
    39 mins ago












  • now it makes sense . 👍
    – Feels awesome
    36 mins ago


















  • so u that mean re-configuring the circuit is the mistake.
    – Feels awesome
    41 mins ago










  • Not at all. It depends what you're doing it for. You lose the detail of the circuit when you simplify, and you can't go on a random path and expect to recreate it.
    – Chu
    39 mins ago












  • now it makes sense . 👍
    – Feels awesome
    36 mins ago
















so u that mean re-configuring the circuit is the mistake.
– Feels awesome
41 mins ago




so u that mean re-configuring the circuit is the mistake.
– Feels awesome
41 mins ago












Not at all. It depends what you're doing it for. You lose the detail of the circuit when you simplify, and you can't go on a random path and expect to recreate it.
– Chu
39 mins ago






Not at all. It depends what you're doing it for. You lose the detail of the circuit when you simplify, and you can't go on a random path and expect to recreate it.
– Chu
39 mins ago














now it makes sense . 👍
– Feels awesome
36 mins ago




now it makes sense . 👍
– Feels awesome
36 mins ago













0














THe upper cct has a bridge current of 12V/4Ω=3A but a total current of 36V /4.5Ω = 8A from (9//9=4.5Ω)



The lower circuit has a bridge current of 0A yet still a total current of 8A.





schematic





simulate this circuit – Schematic created using CircuitLab



These are not the same circuits.






share|improve this answer























  • but hey both are same circuits ,right ?.and we still get two different values of current hiw is it possible?
    – Feels awesome
    2 hours ago










  • THey total current is the same but not the same circuits with the R's swapped and bridged. Once side is 12V the other 24V
    – Tony EE rocketscientist
    2 hours ago










  • and the upper circuit has the current of 3 amp in the bridge .
    – Feels awesome
    2 hours ago










  • typo...........
    – Tony EE rocketscientist
    2 hours ago










  • why arent they equivalent circuits ? I think , I have only rearranged them without any mistakes
    – Feels awesome
    2 hours ago
















0














THe upper cct has a bridge current of 12V/4Ω=3A but a total current of 36V /4.5Ω = 8A from (9//9=4.5Ω)



The lower circuit has a bridge current of 0A yet still a total current of 8A.





schematic





simulate this circuit – Schematic created using CircuitLab



These are not the same circuits.






share|improve this answer























  • but hey both are same circuits ,right ?.and we still get two different values of current hiw is it possible?
    – Feels awesome
    2 hours ago










  • THey total current is the same but not the same circuits with the R's swapped and bridged. Once side is 12V the other 24V
    – Tony EE rocketscientist
    2 hours ago










  • and the upper circuit has the current of 3 amp in the bridge .
    – Feels awesome
    2 hours ago










  • typo...........
    – Tony EE rocketscientist
    2 hours ago










  • why arent they equivalent circuits ? I think , I have only rearranged them without any mistakes
    – Feels awesome
    2 hours ago














0












0








0






THe upper cct has a bridge current of 12V/4Ω=3A but a total current of 36V /4.5Ω = 8A from (9//9=4.5Ω)



The lower circuit has a bridge current of 0A yet still a total current of 8A.





schematic





simulate this circuit – Schematic created using CircuitLab



These are not the same circuits.






share|improve this answer














THe upper cct has a bridge current of 12V/4Ω=3A but a total current of 36V /4.5Ω = 8A from (9//9=4.5Ω)



The lower circuit has a bridge current of 0A yet still a total current of 8A.





schematic





simulate this circuit – Schematic created using CircuitLab



These are not the same circuits.







share|improve this answer














share|improve this answer



share|improve this answer








edited 2 hours ago

























answered 2 hours ago









Tony EE rocketscientist

61.7k22193




61.7k22193












  • but hey both are same circuits ,right ?.and we still get two different values of current hiw is it possible?
    – Feels awesome
    2 hours ago










  • THey total current is the same but not the same circuits with the R's swapped and bridged. Once side is 12V the other 24V
    – Tony EE rocketscientist
    2 hours ago










  • and the upper circuit has the current of 3 amp in the bridge .
    – Feels awesome
    2 hours ago










  • typo...........
    – Tony EE rocketscientist
    2 hours ago










  • why arent they equivalent circuits ? I think , I have only rearranged them without any mistakes
    – Feels awesome
    2 hours ago


















  • but hey both are same circuits ,right ?.and we still get two different values of current hiw is it possible?
    – Feels awesome
    2 hours ago










  • THey total current is the same but not the same circuits with the R's swapped and bridged. Once side is 12V the other 24V
    – Tony EE rocketscientist
    2 hours ago










  • and the upper circuit has the current of 3 amp in the bridge .
    – Feels awesome
    2 hours ago










  • typo...........
    – Tony EE rocketscientist
    2 hours ago










  • why arent they equivalent circuits ? I think , I have only rearranged them without any mistakes
    – Feels awesome
    2 hours ago
















but hey both are same circuits ,right ?.and we still get two different values of current hiw is it possible?
– Feels awesome
2 hours ago




but hey both are same circuits ,right ?.and we still get two different values of current hiw is it possible?
– Feels awesome
2 hours ago












THey total current is the same but not the same circuits with the R's swapped and bridged. Once side is 12V the other 24V
– Tony EE rocketscientist
2 hours ago




THey total current is the same but not the same circuits with the R's swapped and bridged. Once side is 12V the other 24V
– Tony EE rocketscientist
2 hours ago












and the upper circuit has the current of 3 amp in the bridge .
– Feels awesome
2 hours ago




and the upper circuit has the current of 3 amp in the bridge .
– Feels awesome
2 hours ago












typo...........
– Tony EE rocketscientist
2 hours ago




typo...........
– Tony EE rocketscientist
2 hours ago












why arent they equivalent circuits ? I think , I have only rearranged them without any mistakes
– Feels awesome
2 hours ago




why arent they equivalent circuits ? I think , I have only rearranged them without any mistakes
– Feels awesome
2 hours ago











0














Analyzing the circuit by using only Ohm's law and equivalent resistance we can calculate the current passing through each wire in the circuit:



circuit



It is clear that there's a 3A current going from node C to D.



Now if you switched R3 and R4 positions. then indeed no current will flow through the mid wire but the thing is, the current passing through the wire is not i. call it i2 as the circuit has indeed changed.






share|improve this answer





















  • i agree with ur point . but we know that the first circuit(in my qs) is equivalent to the second circuit(in my qs) . so there is most probably no mistake the rearrangement .so where is the mistake ?
    – Feels awesome
    1 hour ago












  • node C and node D dissappeared. so it's not the same circuit.
    – fhlb
    1 hour ago










  • no the node c and D has not disappeared it is present in the last circuit
    – Feels awesome
    59 mins ago










  • 2 new nodes appeared but C and D are clearly note there. C by definition is the node joining R2,R4 and node D
    – fhlb
    53 mins ago










  • thats fine but what is the error in my rearrangement . ?
    – Feels awesome
    51 mins ago
















0














Analyzing the circuit by using only Ohm's law and equivalent resistance we can calculate the current passing through each wire in the circuit:



circuit



It is clear that there's a 3A current going from node C to D.



Now if you switched R3 and R4 positions. then indeed no current will flow through the mid wire but the thing is, the current passing through the wire is not i. call it i2 as the circuit has indeed changed.






share|improve this answer





















  • i agree with ur point . but we know that the first circuit(in my qs) is equivalent to the second circuit(in my qs) . so there is most probably no mistake the rearrangement .so where is the mistake ?
    – Feels awesome
    1 hour ago












  • node C and node D dissappeared. so it's not the same circuit.
    – fhlb
    1 hour ago










  • no the node c and D has not disappeared it is present in the last circuit
    – Feels awesome
    59 mins ago










  • 2 new nodes appeared but C and D are clearly note there. C by definition is the node joining R2,R4 and node D
    – fhlb
    53 mins ago










  • thats fine but what is the error in my rearrangement . ?
    – Feels awesome
    51 mins ago














0












0








0






Analyzing the circuit by using only Ohm's law and equivalent resistance we can calculate the current passing through each wire in the circuit:



circuit



It is clear that there's a 3A current going from node C to D.



Now if you switched R3 and R4 positions. then indeed no current will flow through the mid wire but the thing is, the current passing through the wire is not i. call it i2 as the circuit has indeed changed.






share|improve this answer












Analyzing the circuit by using only Ohm's law and equivalent resistance we can calculate the current passing through each wire in the circuit:



circuit



It is clear that there's a 3A current going from node C to D.



Now if you switched R3 and R4 positions. then indeed no current will flow through the mid wire but the thing is, the current passing through the wire is not i. call it i2 as the circuit has indeed changed.







share|improve this answer












share|improve this answer



share|improve this answer










answered 1 hour ago









fhlb

789721




789721












  • i agree with ur point . but we know that the first circuit(in my qs) is equivalent to the second circuit(in my qs) . so there is most probably no mistake the rearrangement .so where is the mistake ?
    – Feels awesome
    1 hour ago












  • node C and node D dissappeared. so it's not the same circuit.
    – fhlb
    1 hour ago










  • no the node c and D has not disappeared it is present in the last circuit
    – Feels awesome
    59 mins ago










  • 2 new nodes appeared but C and D are clearly note there. C by definition is the node joining R2,R4 and node D
    – fhlb
    53 mins ago










  • thats fine but what is the error in my rearrangement . ?
    – Feels awesome
    51 mins ago


















  • i agree with ur point . but we know that the first circuit(in my qs) is equivalent to the second circuit(in my qs) . so there is most probably no mistake the rearrangement .so where is the mistake ?
    – Feels awesome
    1 hour ago












  • node C and node D dissappeared. so it's not the same circuit.
    – fhlb
    1 hour ago










  • no the node c and D has not disappeared it is present in the last circuit
    – Feels awesome
    59 mins ago










  • 2 new nodes appeared but C and D are clearly note there. C by definition is the node joining R2,R4 and node D
    – fhlb
    53 mins ago










  • thats fine but what is the error in my rearrangement . ?
    – Feels awesome
    51 mins ago
















i agree with ur point . but we know that the first circuit(in my qs) is equivalent to the second circuit(in my qs) . so there is most probably no mistake the rearrangement .so where is the mistake ?
– Feels awesome
1 hour ago






i agree with ur point . but we know that the first circuit(in my qs) is equivalent to the second circuit(in my qs) . so there is most probably no mistake the rearrangement .so where is the mistake ?
– Feels awesome
1 hour ago














node C and node D dissappeared. so it's not the same circuit.
– fhlb
1 hour ago




node C and node D dissappeared. so it's not the same circuit.
– fhlb
1 hour ago












no the node c and D has not disappeared it is present in the last circuit
– Feels awesome
59 mins ago




no the node c and D has not disappeared it is present in the last circuit
– Feels awesome
59 mins ago












2 new nodes appeared but C and D are clearly note there. C by definition is the node joining R2,R4 and node D
– fhlb
53 mins ago




2 new nodes appeared but C and D are clearly note there. C by definition is the node joining R2,R4 and node D
– fhlb
53 mins ago












thats fine but what is the error in my rearrangement . ?
– Feels awesome
51 mins ago




thats fine but what is the error in my rearrangement . ?
– Feels awesome
51 mins ago










Feels awesome is a new contributor. Be nice, and check out our Code of Conduct.










draft saved

draft discarded


















Feels awesome is a new contributor. Be nice, and check out our Code of Conduct.













Feels awesome is a new contributor. Be nice, and check out our Code of Conduct.












Feels awesome is a new contributor. Be nice, and check out our Code of Conduct.
















Thanks for contributing an answer to Electrical Engineering Stack Exchange!


  • Please be sure to answer the question. Provide details and share your research!

But avoid



  • Asking for help, clarification, or responding to other answers.

  • Making statements based on opinion; back them up with references or personal experience.


Use MathJax to format equations. MathJax reference.


To learn more, see our tips on writing great answers.





Some of your past answers have not been well-received, and you're in danger of being blocked from answering.


Please pay close attention to the following guidance:


  • Please be sure to answer the question. Provide details and share your research!

But avoid



  • Asking for help, clarification, or responding to other answers.

  • Making statements based on opinion; back them up with references or personal experience.


To learn more, see our tips on writing great answers.




draft saved


draft discarded














StackExchange.ready(
function () {
StackExchange.openid.initPostLogin('.new-post-login', 'https%3a%2f%2felectronics.stackexchange.com%2fquestions%2f414259%2fwhat-is-the-value-of-the-current-i-and-what-is-the-right-way-to-do-it%23new-answer', 'question_page');
}
);

Post as a guest















Required, but never shown





















































Required, but never shown














Required, but never shown












Required, but never shown







Required, but never shown

































Required, but never shown














Required, but never shown












Required, but never shown







Required, but never shown







Popular posts from this blog

サソリ

広島県道265号伴広島線

Setup Asymptote in Texstudio